LSAT and Law School Admissions Forum

Get expert LSAT preparation and law school admissions advice from PowerScore Test Preparation.

 woodirene
  • Posts: 2
  • Joined: May 13, 2012
|
#4247
Hello!

So I'm having a difficult time accepting the "Uniqueness Theory of Answer Choices" for an answer question explanation of number 19 (Lesson 9 page 9-68).
When I sketched out the question F drives Thursday and Saturday since it can't drive T or W and it can't be consecutive. So the fact that E is the correct answer for the "could be true" stimilus - I'm just not following. I understand A and B are eliminted because of the numerical rules (2-2-1-1), C and D are also eliminated because of other rules. BUT while I was doing the drill and read E, I felt that it wasn't right either because of the rules.

SO -> I'm confused. Help. :hmm: :x :-?
 Adam Tyson
PowerScore Staff
  • PowerScore Staff
  • Posts: 5153
  • Joined: Apr 14, 2011
|
#4252
Hi Irene. With this question we need to draw a mini-diagram that encompasses everything that we know must be true. You got the big inference here, that F drives Thursday and Saturday. So, what else MUST happen? Well, we know that J drives Wednesday or Saturday or both, and could drive other days (the fourth rule). Since F drives Saturday, then J must drive Wednesday. So far we have this:

__ __ J F __ F
M Tu W Th F Sa

So what else do we know? We are in the 2-2-1-1 distribution - good job seeing that. J cannot drive Tuesday, because of the second rule forbidding drivers from driving consecutive days. Everybody has to drive at least one, so in the three remaining spaces we need at least one G and at least one H, and then another G or H or J. Where we put them all is pretty loose at this point - just beware of the no consecutive days rule, right?

A and B are out because of the numerical distribution. C is out because J has to drive Wednesday, a day immediately before F. D is out because J, not G, drives Wednesday. E is left by process of elimination, but we can test it in case you aren't comfortable. Try putting J on Friday - could it work? It looks like this now:

__ __ J F J F

Sure, it's fine - just put G and H in the first two spots, in either order, and you're done! If we can come up with even one solution where J drives on Friday, then that could be true, and in this case we have two solutions for that situation.

Was there a particular rule that you felt conflicted with this? Maybe you weren't clear on J being able to drive other days beside Wednesday or Saturday?

Hope that helps!

Adam M. Tyson
PowerScore LSAT Instructor
 woodirene
  • Posts: 2
  • Joined: May 13, 2012
|
#4289
Hi Adam!!

:lol: I made the mistake of not labeling my template when I drew the mini template and totally spaced on the fact that it was M-Sa not M-F (even though out loud I was saying M-Sa, my mini template was designed M-F- and I didn't catch it in the allotted time of 8:45mins). So - I kept thinking that F and F were on W and F - my mistake was further heightened with the "F" being on the last one of my faulty mini template.

UGH - Mental tiredness. THANKS though! And I apologize for the rather silly question given my oversight.

Get the most out of your LSAT Prep Plus subscription.

Analyze and track your performance with our Testing and Analytics Package.